Last visit was: 24 Apr 2024, 15:08 It is currently 24 Apr 2024, 15:08

Close
GMAT Club Daily Prep
Thank you for using the timer - this advanced tool can estimate your performance and suggest more practice questions. We have subscribed you to Daily Prep Questions via email.

Customized
for You

we will pick new questions that match your level based on your Timer History

Track
Your Progress

every week, we’ll send you an estimated GMAT score based on your performance

Practice
Pays

we will pick new questions that match your level based on your Timer History
Not interested in getting valuable practice questions and articles delivered to your email? No problem, unsubscribe here.
Close
Request Expert Reply
Confirm Cancel
SORT BY:
Kudos
e-GMAT Representative
Joined: 02 Nov 2011
Posts: 4346
Own Kudos [?]: 30782 [38]
Given Kudos: 635
GMAT Date: 08-19-2020
Send PM
e-GMAT Representative
Joined: 02 Nov 2011
Posts: 4346
Own Kudos [?]: 30782 [4]
Given Kudos: 635
GMAT Date: 08-19-2020
Send PM
e-GMAT Representative
Joined: 02 Nov 2011
Posts: 4346
Own Kudos [?]: 30782 [1]
Given Kudos: 635
GMAT Date: 08-19-2020
Send PM
Retired Moderator
Joined: 10 May 2010
Posts: 796
Own Kudos [?]: 628 [0]
Given Kudos: 192
Send PM
Re: A primer on Variance Analysis [#permalink]
Not sure about F. If not thanked members are as likely as thanked members then this proves that thanks has no significance in GMAT score. Vice versa proves that thanks has significance in GMAT score.
e-GMAT Representative
Joined: 02 Nov 2011
Posts: 4346
Own Kudos [?]: 30782 [0]
Given Kudos: 635
GMAT Date: 08-19-2020
Send PM
Re: A primer on Variance Analysis [#permalink]
Expert Reply
AbhiJ wrote:
Not sure about F. If not thanked members are as likely as thanked members then this proves that thanks has no significance in GMAT score. Vice versa proves that thanks has significance in GMAT score.


Let’s understand the argument to address your doubt.

Joshua is one of the most thanked members on GMAT Club. Hence, Joshua will most likely score above 700 on his GMAT.

Let’s understand the conclusion. The conclusion states that Joshua has greater than 50% (note: most likely = greater than 50%) chance of scoring above 700. For the sake of this discussion, to make things simple, let’s fix the conclusion at 60%. So our modified argument will be:

Joshua is one of the most thanked members on GMAT Club. Hence, there is 60% probability that Joshua will score about 700.

Note: The conclusion is about a specific likelihood.

Now let’s take choice F
• Whether GMAT Club members who are not rated “most thanked” are as likely to score above 700 as those who are?

Extreme 1: Yes, GMAT Club members who are not rated “most thanked” are as likely to score above 700 as those who are?
Does Extreme 1 validate/invalidate the hypothesis that there is a 60% probability that Joshua will score above 700?
Not at all, since we are not discussing any probability number here. Furthermore, as stated earlier, the argument is about achieving a specific outcome. The performance of a complimentary segment does not have a bearing on the conclusion.
Carefully note that the argument does not say and neither should you assume that being the “most thanked” member is the only way to get a score of 700.

Extreme 2: No, GMAT Club members who are not rated “most thanked” are as likely to score above 700 as those who are?
Does Extreme 2 validate/invalidate the hypotheses that there is a 60% probability that Joshua will score above 700?
The discussion of Extreme 1 applies here as well. This extreme does not validate or invalidate the conclusion since we are not talking about any probability number here.

Hope this helps clarify your doubt.

-Rajat
User avatar
Manager
Manager
Joined: 02 Sep 2012
Posts: 161
Own Kudos [?]: 568 [0]
Given Kudos: 99
Location: United States
Concentration: Entrepreneurship, Finance
GMAT Date: 07-25-2013
GPA: 3.83
WE:Architecture (Computer Hardware)
Send PM
Re: A primer on Variance Analysis [#permalink]
E-Gmat,

Is E the answer to the last question mentioned in the post?
e-GMAT Representative
Joined: 02 Nov 2011
Posts: 4346
Own Kudos [?]: 30782 [0]
Given Kudos: 635
GMAT Date: 08-19-2020
Send PM
Re: A primer on Variance Analysis [#permalink]
Expert Reply
skamal7 wrote:
E-Gmat,

Is E the answer to the last question mentioned in the post?


Hi,

Yes. The answer is E. :)

Thanks,
Chiranjeev
avatar
Intern
Intern
Joined: 21 Oct 2009
Posts: 11
Own Kudos [?]: 1 [1]
Given Kudos: 0
Send PM
Re: A primer on Variance Analysis [#permalink]
1
Bookmarks
Hi Chiranjeev,

Could you please explain why ans 'E' is correct and others are not?

Thanks for posting the article. This is my 1st attempt at answering such questions, so feel free to point out mistakes in my logic & reasoning.

The argument states that Nile Delta was invaded/ruled 1650-1550 by Hyskos and that these people maybe Canaanites. Reason – excavations at Avaris (which was Hyksos city) showed artifacts similar to those produced in Ashkelon (which was Cannan city)


A. Whether there were some artifacts found at Avaris that were unlike those produced in Ashkelon but that date to before 1700 BC.
Ext 1 – Yes, there were some artifacts found at Avaris that were unlike those produced in Ashkelon but that date to before 1700 BC. This weakens the argument, and it talks about period before 1700BC. The argument is for 1650-1550
Ext-2 – No, there were no artifacts found at Avaris that were unlike those produced in Ashkelon but that date to before 1700 BC. This again talks about time period outside the one discussed in the argument

B. Whether the Hyksos ruled any other part of Egypt besides the Nile Delta in the period from 1650 B.C. to 1550 B.C.
Ext 1 – Yes, Hyksos ruled other part of Egypt besides the Nile Delta in the period from 1650 B.C. to 1550 B.C. This does not validates the argument, perhaps a strengthener, if at all
Ext-2 – No, Hyksos did not rule other part of Egypt besides the Nile Delta in the period from 1650 B.C. to 1550 B.C. This could be a strengthener but does not validate or invalidates if they were Canaanites

C. Whether Avaris was the nearest Hyksos city in Egypt to Canaan.
Ext 1 – Yes, Avaris was the nearest Hyksos city in Egypt to Canaan. Just for the fact that it was the nearest city does not validate or invalidates that Hyksos were Canaanites
Ext-2 – No, Avaris was the nearest Hyksos city in Egypt to Canaan. Same reasoning as above

D. Whether Ashkelon after 1550 B.C. continued to produce artifacts similar to those found at Avaris.
Ext 1 – Yes, Ashkelon continued to produce….Not the correct choice as the argument is not concerned with Ashkelon producing artifacts after 1550
Ext 2 - No, Ashkelon did not continue to produce….same reasoning, not concerned with whether Ashkelon produced after 1550 or not

E. Whether many of the artifacts found at Avaris that are similar to the artifacts produced in Ashkelon date to well before the Hyksos invasion
Ext 1 – Yes, many of the artifacts found at Avaris that are similar to the artifacts produced in Ashkelon date to well before the Hyksos invasion. This validates that Hyksos were producing artifacts similar to those found in Ashkleon way before the invasion and carried through during the invasion as well.
Ext 2 - No, not many of the artifacts found at Avaris that are similar to the artifacts produced in Ashkelon date to well before the Hyksos invasion. Not sure how this invalidates the argument – my guess – the Hyksos produced artifacts perhaps during or after the invasion? Thus not presenting any relation to them being Canaanites?
User avatar
Manager
Manager
Joined: 01 Jun 2010
Status:#TeamFuqua
Posts: 114
Own Kudos [?]: 38 [0]
Given Kudos: 205
Location: United States
Concentration: General Management, Strategy
GMAT 1: 580 Q37 V32
GMAT 2: 570 Q36 V32
GMAT 3: 680 Q44 V40
WE:Consulting (Consulting)
Send PM
Re: A primer on Variance Analysis [#permalink]
Hi Chiranjeev,

Based on this article and the course, I tried applying the variance analysis on this GMATPrep question below and am stumped. Can you help?

Last August the XT chain of gasoline stations had a temporary sales promotion in effect. In the promotion, any customer who made a purchase of ten or more gallons of gasoline was entitled to a free car wash. For the month of August, XT experienced a ten percent increase in gasoline sales as compared to sales in August the previous year, so evidently the promotion was successful as a means of boosting sales.
In evaluating the argument, it would be most helpful to answer which of the following?
A. In the areas in which XT’s gasoline stations operate, how did total combined gasoline sales for all gasoline stations last August compare with sales for the previous August?
B. Was the money that XT earned from the increase in gasoline sales enough to offset the cost of providing free car washes during the promotion?
C. Were there any customers who bought ten or more gallons at an XT gasoline station during the promotion who would have or more gallons at an XT gasoline in lower quantities, but more frequently, if the promotion had not been in effect?
D. Did XT or any of its gasoline stations have to pay other businesses to provide the car washes that customers were offered in the promotion?
E. Are XT’s gasoline sales in August usually significantly higher than one twelfth of XT’s annual gasoline sales?

My selected answer was C because if more number of people who would normally purchase gasoline in amounts <10 Gal were now purchasing >10 Gal due to the promotion, then that could result in the promotion being successful. If I apply the variance test as follows
More people chose to purchase gasoline >10 Gal because of promotion who would otherwise have not - Validates conclusion -> promotion was successful.
People who purchased > 10 Gal gasoline did not intend to purchase gasoline in smaller amounts than 10 Gal than before ==> these are not new,converted customers due to the promotion - Invalidates conclusion ==> promotion not successful in bringing new customer who contributed to the increased sales.

Now, per the variance test this option holds true. But the OA is A. :| Can you help me think about how to solve this question and help outline an approach? Thanks!
e-GMAT Representative
Joined: 02 Nov 2011
Posts: 4346
Own Kudos [?]: 30782 [0]
Given Kudos: 635
GMAT Date: 08-19-2020
Send PM
Re: A primer on Variance Analysis [#permalink]
Expert Reply
MBAjunkie16 wrote:
Hi Chiranjeev,

Based on this article and the course, I tried applying the variance analysis on this GMATPrep question below and am stumped. Can you help?

Last August the XT chain of gasoline stations had a temporary sales promotion in effect. In the promotion, any customer who made a purchase of ten or more gallons of gasoline was entitled to a free car wash. For the month of August, XT experienced a ten percent increase in gasoline sales as compared to sales in August the previous year, so evidently the promotion was successful as a means of boosting sales.
In evaluating the argument, it would be most helpful to answer which of the following?
A. In the areas in which XT’s gasoline stations operate, how did total combined gasoline sales for all gasoline stations last August compare with sales for the previous August?
B. Was the money that XT earned from the increase in gasoline sales enough to offset the cost of providing free car washes during the promotion?
C. Were there any customers who bought ten or more gallons at an XT gasoline station during the promotion who would have or more gallons at an XT gasoline in lower quantities, but more frequently, if the promotion had not been in effect?
D. Did XT or any of its gasoline stations have to pay other businesses to provide the car washes that customers were offered in the promotion?
E. Are XT’s gasoline sales in August usually significantly higher than one twelfth of XT’s annual gasoline sales?

My selected answer was C because if more number of people who would normally purchase gasoline in amounts <10 Gal were now purchasing >10 Gal due to the promotion, then that could result in the promotion being successful. If I apply the variance test as follows
More people chose to purchase gasoline >10 Gal because of promotion who would otherwise have not - Validates conclusion -> promotion was successful.
People who purchased > 10 Gal gasoline did not intend to purchase gasoline in smaller amounts than 10 Gal than before ==> these are not new,converted customers due to the promotion - Invalidates conclusion ==> promotion not successful in bringing new customer who contributed to the increased sales.

Now, per the variance test this option holds true. But the OA is A. :| Can you help me think about how to solve this question and help outline an approach? Thanks!


Hi,

First of all, option C is grammatically incorrect. As is, the option statement does not make any sense to me. I even asked my colleagues to help me out and they were also clueless :!:

Now, even if I don't understand option C completely, I can still see that it is incorrect. When you analyse this option statement, you have ignored "were there any" part in the option. So, if you look at it: the option C wants to find out if there is at least one customer who would have something something. Now, does that provide any relevant information?

The answer is a BIG NO. What do we gain by knowing that there is at least one sweet guy out there who would have done something. In a population, you will have find some people doing one thing or the other. It does not really impact the conclusion about the success of the program as a whole. You can appreciate that a single person cannot really affect the success of a program. Right?

Now, just to tell you - this is a common trick used by GMAT in not only evaluate but also strengthen and weaken questions, where they will talk about a general trend in the argument and one of the option statement will says that "some" people did not follow the trend. This does not mean that general trend does not hold because "some" persons don't follow it. There are always going to be exceptions.

For example: Consider this official question and let me know the correct answer:

Businesses are suffering because of a lack of money available for development loans. To help businesses, the government plans to modify the income-tax structure in order to induce individual taxpayers to put a larger portion of their incomes into retirement savings accounts, because as more money is deposited in such accounts, more money becomes available to borrowers.

Which of the following, if true, raises the most serious doubt regarding the effectiveness of the government’s plan to increase the amount of money available for development loans for businesses?

(A) When levels of personal retirement savings increase, consumer borrowing always increases correspondingly.
(B) The increased tax revenue the government would receive as a result of business expansion would not offset the loss in revenue from personal income taxes during the first year of the plan.
(C) Even with tax incentives, some people will choose not to increase their levels of retirement savings.
(D) Bankers generally will not continue to lend money to businesses whose prospective earnings are insufficient to meet their loan repayment schedules.
(E) The modified tax structure would give all taxpayers, regardless of their incomes, the same tax savings for a given increase in their retirement savings.

Thanks,
Chiranjeev
User avatar
Manager
Manager
Joined: 01 Jun 2010
Status:#TeamFuqua
Posts: 114
Own Kudos [?]: 38 [0]
Given Kudos: 205
Location: United States
Concentration: General Management, Strategy
GMAT 1: 580 Q37 V32
GMAT 2: 570 Q36 V32
GMAT 3: 680 Q44 V40
WE:Consulting (Consulting)
Send PM
Re: A primer on Variance Analysis [#permalink]
Quote:
Hi,

First of all, option C is grammatically incorrect. As is, the option statement does not make any sense to me. I even asked my colleagues to help me out and they were also clueless :!:


My bad - I copied it from an online location since GMATPrep software doesn't allow you to copy paste and didn't realize its missing a few key words.

Were there any customers who bought ten or more gallons at an XT gasoline station during the promotion who would have purchased 10 or more gallons at an XT gasoline in lower quantities, but more frequently, if the promotion had not been in effect?

Quote:
When you analyse this option statement, you have ignored "were there any" part in the option. So, if you look at it: the option C wants to find out if there is at least one customer who would have something something. Now, does that provide any relevant information?

The answer is a BIG NO. What do we gain by knowing that there is at least one sweet guy out there who would have done something. In a population, you will have find some people doing one thing or the other. It does not really impact the conclusion about the success of the program as a whole. You can appreciate that a single person cannot really affect the success of a program. Right?

Now, just to tell you - this is a common trick used by GMAT in not only evaluate but also strengthen and weaken questions, where they will talk about a general trend in the argument and one of the option statement will says that "some" people did not follow the trend. This does not mean that general trend does not hold because "some" persons don't follow it. There are always going to be exceptions.


This is brilliant and my biggest takeaway. Thanks for explaining it so clearly. :-D
Quote:
For example: Consider this official question and let me know the correct answer:

Businesses are suffering because of a lack of money available for development loans. To help businesses, the government plans to modify the income-tax structure in order to induce individual taxpayers to put a larger portion of their incomes into retirement savings accounts, because as more money is deposited in such accounts, more money becomes available to borrowers.

Which of the following, if true, raises the most serious doubt regarding the effectiveness of the government’s plan to increase the amount of money available for development loans for businesses?

(A) When levels of personal retirement savings increase, consumer borrowing always increases correspondingly.
(B) The increased tax revenue the government would receive as a result of business expansion would not offset the loss in revenue from personal income taxes during the first year of the plan.
(C) Even with tax incentives, some people will choose not to increase their levels of retirement savings.
(D) Bankers generally will not continue to lend money to businesses whose prospective earnings are insufficient to meet their loan repayment schedules.
(E) The modified tax structure would give all taxpayers, regardless of their incomes, the same tax savings for a given increase in their retirement savings.

Thanks,
Chiranjeev


In this case, my answer is A. And the thought process behind it as follows:

Two weakeners for the argument can be potentially
a) Money saved due to the changed tax structure and consequent savings does not go towards disbursing development loans
b) Despite govt encouragement, people don't save more in retirement savings accounts

Based on this both A and C were potential answers. When applying the variance analysis on C, and taking into consideration that it says "some people" we can eliminate it.

Please let me know if this the right way. Thanks!
e-GMAT Representative
Joined: 02 Nov 2011
Posts: 4346
Own Kudos [?]: 30782 [0]
Given Kudos: 635
GMAT Date: 08-19-2020
Send PM
Re: A primer on Variance Analysis [#permalink]
Expert Reply
MBAjunkie16 wrote:
Quote:
Hi,

First of all, option C is grammatically incorrect. As is, the option statement does not make any sense to me. I even asked my colleagues to help me out and they were also clueless :!:


My bad - I copied it from an online location since GMATPrep software doesn't allow you to copy paste and didn't realize its missing a few key words.

Were there any customers who bought ten or more gallons at an XT gasoline station during the promotion who would have purchased 10 or more gallons at an XT gasoline in lower quantities, but more frequently, if the promotion had not been in effect?

Quote:
When you analyse this option statement, you have ignored "were there any" part in the option. So, if you look at it: the option C wants to find out if there is at least one customer who would have something something. Now, does that provide any relevant information?

The answer is a BIG NO. What do we gain by knowing that there is at least one sweet guy out there who would have done something. In a population, you will have find some people doing one thing or the other. It does not really impact the conclusion about the success of the program as a whole. You can appreciate that a single person cannot really affect the success of a program. Right?

Now, just to tell you - this is a common trick used by GMAT in not only evaluate but also strengthen and weaken questions, where they will talk about a general trend in the argument and one of the option statement will says that "some" people did not follow the trend. This does not mean that general trend does not hold because "some" persons don't follow it. There are always going to be exceptions.


This is brilliant and my biggest takeaway. Thanks for explaining it so clearly. :-D
Quote:
For example: Consider this official question and let me know the correct answer:

Businesses are suffering because of a lack of money available for development loans. To help businesses, the government plans to modify the income-tax structure in order to induce individual taxpayers to put a larger portion of their incomes into retirement savings accounts, because as more money is deposited in such accounts, more money becomes available to borrowers.

Which of the following, if true, raises the most serious doubt regarding the effectiveness of the government’s plan to increase the amount of money available for development loans for businesses?

(A) When levels of personal retirement savings increase, consumer borrowing always increases correspondingly.
(B) The increased tax revenue the government would receive as a result of business expansion would not offset the loss in revenue from personal income taxes during the first year of the plan.
(C) Even with tax incentives, some people will choose not to increase their levels of retirement savings.
(D) Bankers generally will not continue to lend money to businesses whose prospective earnings are insufficient to meet their loan repayment schedules.
(E) The modified tax structure would give all taxpayers, regardless of their incomes, the same tax savings for a given increase in their retirement savings.

Thanks,
Chiranjeev


In this case, my answer is A. And the thought process behind it as follows:

Two weakeners for the argument can be potentially
a) Money saved due to the changed tax structure and consequent savings does not go towards disbursing development loans
b) Despite govt encouragement, people don't save more in retirement savings accounts

Based on this both A and C were potential answers. When applying the variance analysis on C, and taking into consideration that it says "some people" we can eliminate it.

Please let me know if this the right way. Thanks!

Yes. Absolutely correct.

-Chiranjeev
Math Expert
Joined: 02 Sep 2009
Posts: 92900
Own Kudos [?]: 618806 [0]
Given Kudos: 81588
Send PM
Re: A primer on Variance Analysis [#permalink]
Expert Reply

Included in our Best Downloads of GMAT Club

User avatar
Non-Human User
Joined: 01 Oct 2013
Posts: 17213
Own Kudos [?]: 848 [0]
Given Kudos: 0
Send PM
Re: A primer on Variance Analysis [#permalink]
Hello from the GMAT Club VerbalBot!

Thanks to another GMAT Club member, I have just discovered this valuable topic, yet it had no discussion for over a year. I am now bumping it up - doing my job. I think you may find it valuable (esp those replies with Kudos).

Want to see all other topics I dig out? Follow me (click follow button on profile). You will receive a summary of all topics I bump in your profile area as well as via email.
GMAT Club Bot
Re: A primer on Variance Analysis [#permalink]
Moderators:
GMAT Club Verbal Expert
6920 posts
GMAT Club Verbal Expert
238 posts
CR Forum Moderator
832 posts

Powered by phpBB © phpBB Group | Emoji artwork provided by EmojiOne